Difference between revisions of "2005 IMO Shortlist Problems/A2"
5849206328x (talk | contribs) m (→Solution 1) |
|||
Line 12: | Line 12: | ||
=== Solution 1 === | === Solution 1 === | ||
− | '''Lemma 1.''' <math> | + | '''Lemma 1.''' <math>f </math> is non-decreasing. |
− | ''Proof.'' Suppose, on the contrary, that there exist <math> | + | ''Proof.'' Suppose, on the contrary, that there exist <math>x> z</math> such that <math>f(x) < f(z) </math>. We set <math>y = \frac{x-z}{f(z)-f(x)} > 0 </math>, so <math>x + yf(x) = z + yf(z) </math>, to obtain |
<center> | <center> | ||
− | <math> | + | <math>f(x)f(y) = 2f(x+yf(x)) = 2f(z+yf(z)) = f(z)f(y) </math>, |
</center> | </center> | ||
− | or <math> | + | or <math>f(x) = f(z) </math>, a contradiction. {{Halmos}} |
− | '''Lemma 2.''' <math> | + | '''Lemma 2.''' <math>f </math> is not strictly increasing. |
− | ''Proof.'' Assume the contrary. Then for all <math> | + | ''Proof.'' Assume the contrary. Then for all <math>x ,y </math> we have |
− | <center> <math> | + | <center> <math>f(x)f(y) = 2f(x + yf(x)) > 2f(x) </math>, </center> |
− | so <math> | + | so <math>f(y) > 2 </math>. Furthermore, since <math>f </math> is [[injective]], we have <math>f(x+1 \cdot f(x)) = f(x)f(1) = f(1+ x\cdot f(1)) </math>, which implies <math>x+ f(x) = 1+ x \cdot f(1)) </math>, or |
− | <center> <math> | + | <center> <math>f(x) = (f(1)-1)x + 1 </math>. </center> |
− | But then for <math> | + | But then for <math>x </math> arbitrarily close to 0, <math>f(x) </math> becomes less than 2, a contradiction. Thus <math>f </math> is not strictly increasing. {{Halmos}} |
− | Now, let <math> | + | Now, let <math>x>z </math>, <math>f(x) = f(z) </math>. If <math>y </math> is in the interval <math>(0, (x-z)/f(x) </math>, then <math> z< z+yf(z) \le x </math>, so |
<center> | <center> | ||
<math> f(z) \le f(z+yf(z)) \le f(x) = f(z) </math>, </center> | <math> f(z) \le f(z+yf(z)) \le f(x) = f(z) </math>, </center> | ||
− | so <math> | + | so <math>f(x+yf(z)) = f(x) </math>. It follows that |
<center> | <center> | ||
− | <math> | + | <math>f(z)f(y) = 2f(z + yf(z)) = 2f(x) = 2f(z) </math>, |
</center> | </center> | ||
− | so <math> | + | so <math>f(y) = 2 </math>, for all <math>y </math> in that interval. |
− | |||
− | |||
− | |||
+ | But if <math> {} f(y_0) = 2 </math>, then setting <math>x=y=y_0 </math> in the original equation gives <math>f(3^n y_0) = 2 </math>, by induction. It follows that for any <math>x </math>, there exist <math>a<x </math> and <math>c>x </math> such that <math>f(a) = f(c) = 2 </math>, and since <math>f </math> is non-decreasing, we must have <math>f(x) = 2 </math> for all <math>x </math>. It is easy to see that this satisfies the given equation. Q.E.D. | ||
=== Solution 2 === | === Solution 2 === |
Latest revision as of 19:59, 28 March 2010
Problem
(Bulgaria) Let denote the set of all postive real numbers. Determine all functions such that
for all positive real numbers and .
Solutions
Solution 1
Lemma 1. is non-decreasing.
Proof. Suppose, on the contrary, that there exist such that . We set , so , to obtain
,
or , a contradiction. ∎
Lemma 2. is not strictly increasing.
Proof. Assume the contrary. Then for all we have
so . Furthermore, since is injective, we have , which implies , or
But then for arbitrarily close to 0, becomes less than 2, a contradiction. Thus is not strictly increasing. ∎
Now, let , . If is in the interval , then , so
so . It follows that
,
so , for all in that interval.
But if , then setting in the original equation gives , by induction. It follows that for any , there exist and such that , and since is non-decreasing, we must have for all . It is easy to see that this satisfies the given equation. Q.E.D.
Solution 2
Lemma 1. There exist distinct positive such that .
Proof. Suppose the contrary, i.e., suppose is injective. Then for any , we have
,
which implies
.
This means that either and for all (a contradiction, since that is not injective), or , for some real . But setting then gives us a quadratic in with a nonzero leading coefficient, which has at most two real roots, implying that can only assume two different values, a contradiction. ∎
Lemma 2. There exist and infinitely many such that , for all nonnegative integers .
Proof. Let be the distinct positive reals of Lemma 1 such that ; without loss of generality, let . Letting yields
Since , this implies . We now prove that for all nonnegative , by induction. We have just proven our base case. Now, assume . Setting gives us
,
so , as desired. ∎
Lemma 3. For all , .
Proof. We note that is the solution to the equation , which is positive when , so if this is the case, setting to this value gives us
,
and since , this implies, , a contradiction. ∎
Lemma 4. .
Proof. We will prove by induction that , for all . Our base case comes from Lemma 3. Now, if for all , , then for some ,
so , as desired.
Now, suppose there exists some . Then there exists some such that . But this gives us , a contradiction. Thus for all , . ∎
We will now prove that is the only solution to the functional equation. Consider any value of . By Lemma 2, there exists some such that . Setting then gives us
But from Lemma 4, we know , so we must have . This constant function clearly satisfies the given equation. Q.E.D.
Alternate solutions are always welcome. If you have a different, elegant solution to this problem, please add it to this page.